LSAT and Law School Admissions Forum

Get expert LSAT preparation and law school admissions advice from PowerScore Test Preparation.

 maximbasu
  • Posts: 59
  • Joined: May 19, 2016
|
#25546
Hello,
I chose B as the correct answer while the correct answer was E.

I don't understand how E builds off of the stimulus. The stimulus is not focusing on the company's integrity. B captures the idea better; it rejects Tours's claims and that is exactly what the stimulus is doing.

Thank you, Maxim.
 Adam Tyson
PowerScore Staff
  • PowerScore Staff
  • Posts: 5153
  • Joined: Apr 14, 2011
|
#25592
Read that stimulus again, and focus on the conclusion - the author is acknowledging that all of Tours' evidence supports the idea of a raise, but he still concludes that she should not get one. His only premise in support of that conclusion is that giving her a raise would "jeopardize the integrity of the firm’s merit based reward system." The integrity of the firm is exactly what we need to focus on, because it is the only premise offered in support of the conclusion.

B does not accurately describe what happened in the stimulus because at no point does the author do anything to undermine the persuasiveness of Tours' evidence. He is in complete agreement with all of her evidence, but objects to the raise on completely different grounds. He is not saying that she is "merely complaining" - he is saying she is absolutely correct, but that it would nonetheless be bad for the company to comply with her request.
 adlindsey
  • Posts: 90
  • Joined: Oct 02, 2016
|
#38102
Me and Maxim must be on the same frequency, because this is third question we answer the same. I also had B. So what role does "complaining" play in the argument. I don't see how E, "overlooks implications" of integrity or what that means.
User avatar
 Dave Killoran
PowerScore Staff
  • PowerScore Staff
  • Posts: 5853
  • Joined: Mar 25, 2011
|
#38253
adlindsey wrote:Me and Maxim must be on the same frequency, because this is third question we answer the same. I also had B. So what role does "complaining" play in the argument. I don't see how E, "overlooks implications" of integrity or what that means.
Hi A,

"Complaining" doesn't actually play a role in the argument. Answer choice (B) is wrong, and thus we don't have to justify what is stated there as being accurate. The Director of Personnel didn't characterize it as "mere complaining"—reasons were given (albeit bad ones) for turning down her request—which is why the answer is incorrect.

Adam's explanation is spot on above, but let me add a few points about the phrase "overlooks the implications." First, if you encounter a phrase that doesn't make sense, it's far more helpful for you to stop and determine what it means on your own than it is for me or anyone else to tell you the meaning. Moments of self-illumination are far brighter than when someone externally shines a light, mainly because it's all you on the test and you can't access outside help. Second, "overlooks the implications" is just a fancy way of saying "ignores the consequences." The people who make this test have repeatedly indicated they expect test takers to be able to analyze and understand language of this sort, and so as we both know, there are many instances of them suddenly speaking in a very high-toned, almost extra-intellectual manner. They do this precisely to throw people off-guard, so you have to be ready for it! Otherwise you fall prey to their trap. Last, what is the Director ignoring the consequences of? How denying Ms. Tours request would affect the integrity of the firm’s merit based reward system. In other words, he's ignoring what the denial means for the integrity of the system. And what it means is that the system is worthless, because the Director has admitted that "her job performance has been both excellent in itself and markedly superior to that of others" and that "Her complaint that she was treated unfairly thus appears justified." So, she's a superior employee who was treated unfairly—shouldn't she get the raise? In a merit-based system, the answer is a resounding yes. But instead, the Director says the answer is no because you don't want to reward complaining. In doing so, the Director indicates that the "merit-based" system is not in fact based entirely on merit, which undermines the integrity of such a system.

Definitely a hard question (under 50% answer it correctly), but another great example of how the test makers think, and what we can learn from studying each question closely. As always, focus on how they think—that's the key here.

Thanks!
 sodomojo
  • Posts: 24
  • Joined: Aug 01, 2017
|
#40862
Adam Tyson wrote:His only premise in support of that conclusion is that giving her a raise would "jeopardize the integrity of the firm’s merit based reward system."
But if this only premise in support of his conclusion consequently has nothing to do with the "superior job performance" he previously mentions, wouldn't that make (C) "sidesteps the issue of whether superior job performance is a suitable basis for awarding salary increases" correct?

Or is it the case that the entire first half of the stimulus discussing her excellent/superior job performance should also be seen as premises of the argument, but merely as premises that don't properly lead to the conclusion?
User avatar
 Dave Killoran
PowerScore Staff
  • PowerScore Staff
  • Posts: 5853
  • Joined: Mar 25, 2011
|
#40869
sodomojo wrote:Or is it the case that the entire first half of the stimulus discussing her excellent/superior job performance should also be seen as premises of the argument, but merely as premises that don't properly lead to the conclusion?
Hi Sodo,

Yes, the first part here can't be ignored (and keep in mind that in many instances premises aren't central to an argument but often allow us to better understand the entire discussion). In this case, the first half actually provides basis for her to receive the raise, since it explicitly indicates there is "compelling evidence that her job performance has been both excellent in itself and markedly superior to that of others in her department who were awarded merit raises. Her complaint that she was treated unfairly thus appears justified." Therefore, with (C), the Director doesn't sidestep the issue, but has instead acknowledged that the job she did warranted a raise.

What Adam is saying is that the Director's only premise in direct support of the conclusion that a raise should be denied is that it jeopardizes the system. It doesn't mean that the other info isn't relevant.

Thanks!
 Pragmatism
  • Posts: 68
  • Joined: Jan 11, 2018
|
#43029
I chose the wrong answer as well, but since this was a very hard question allow me to give my analysis on the entirety of the answer choices. By the way, I chose answer choice C for the same reason mentioned by Sodomojo.

A: "could be handled on an unofficial basis." That would possibly concede that the Director of personnel would go against the very premise that he/she uses to support his/her conclusion.

B: If they did try to undermine Ms. Tours's evidence as "mere complaining," it still doesn't warrant the persuasive evidence that was presented forth. If anything, this would undermine the integrity of a merit based system, by discrediting an excellent performance on the basis of personal appeal to character.

C: This goes inline with answer choice B. Ask yourself, why did the Director of personnel not honor the merit-based system? Was it lack of performance? Certainly not. Then? It was the simple fact that Ms. Tours complained and bought the issue that could possibly undermine the merit-based system to the forefront.

D: This could be true, but nothing of substance is said of some of the people who did receive the raises. The only thing we know is that Ms. Tours "job performance has been both excellent in itself and markedly superior to that of others in her department who were awarded merit raises."

E: This threw me off because it is the only premise that is presented by the Director of personnel to deny Ms. Tours the merit-based raise. But in order to accept the conclusion, "Ms. Tours request should be denied," we cannot accept a premise that undermines itself to support a different judgement (conclusion).

Hope that helps.
 Claire Horan
PowerScore Staff
  • PowerScore Staff
  • Posts: 408
  • Joined: Apr 18, 2016
|
#43225
Hi Pragmatism,

I like how you explained E, and I'll add a gloss in my own words in case anyone out there is still confused.

The argument proceeds by giving reasons why Ms. Tours is entitled to a raise and why her request should be granted. Then the argument shifts abruptly and the director gives only one reason to justify denial of Ms. Tours's request: "To raise Ms. Tours’s salary because of her complaint would jeopardize the integrity of the firm’s merit based reward system." Because this is the ultimate reason that leads to the director's conclusion, we need to attack this reasoning. Answer choice E is the only choice that addresses this reason, which is the basis for the director's conclusion.
User avatar
 Snomen
  • Posts: 35
  • Joined: Sep 30, 2021
|
#94283
Correct me if I am wrong, I had two options A or E..I choose A because I failed to remind myself that stimulus says "official response is required" so knowing this part from the Stimulus would destroy answer choice A(Hope I am right in this reasoning). Second, the problem with E that I encountered was that I thought maybe there are other reasons for the firm to deny raises for Ms. Tour, and because there were other reasons to deny the raise it would not hurt the integrity of the firm.
Thank you in advance!
User avatar
 Beth Hayden
PowerScore Staff
  • PowerScore Staff
  • Posts: 123
  • Joined: Sep 04, 2021
|
#94426
Hi Snomen,

Actually, the stimulus does indicate that she should have gotten the raise by saying her claim of being treated unfairly is justified (i.e., it was unfair to not give her the raise because she deserved it). That means if they don't give her the raise now, they will be knowingly treating her unfairly, which compromises the integrity of the firm's system.

You're right that the stimulus says an official response is required, though I think there is enough room for interpretation on that statement that I would be hesitant to eliminate (A) on that alone. However, there's another big problem. If they handle the issue unofficially, they may find a way of getting her more money, but that does nothing to bolster the integrity of the merit-raise system. The system is still unfair, even if Ms. Tours is satisfied with whatever the company does "unofficially."

Hope that helps!
Beth

Get the most out of your LSAT Prep Plus subscription.

Analyze and track your performance with our Testing and Analytics Package.